LSAT and Law School Admissions Forum

Get expert LSAT preparation and law school admissions advice from PowerScore Test Preparation.

 Administrator
PowerScore Staff
  • PowerScore Staff
  • Posts: 8916
  • Joined: Feb 02, 2011
|
#40588
Complete Question Explanation
(The complete setup for this game can be found here: lsat/viewtopic.php?t=11817)

The correct answer choice is (B)

The question stem asks for what Cannot Be True. Each of the five answer choices lists two lectures given at specified lecture locations. Thus, you should expect that the correct answer will violate the rules about the locations.

From the third rule, exactly two of the lectures are given at H, and from the second rule one of those two lectures is the fourth lecture. Thus, answer choice (B) cannot occur because then three lectures would be given at H, and it is only possible for two lectures to be given there. Thus, answer choice (B) is the correct answer.

Get the most out of your LSAT Prep Plus subscription.

Analyze and track your performance with our Testing and Analytics Package.